« first day (1672 days earlier)      last day (3332 days later) » 

7:00 PM
The floor function just "chops off the decimal" part (essentially the same as "ignoring all but the first digit")
So, if we know $\log_{10}(10^n) = n$, then together with the fact that $\log_{10}$ is increasing, we know that the floor function will return the same integer until $\log_{10}(x)$ reaches $n+1$.
And since $\log_{10}$ is increasing, the FIRST place that happens is at $10^{n+1}$
Which is also the next time we go "up another digit"
Let's look at it another way: $367 = 3.67 \times 10^2$
So $\log_{10}(367) = \log_{10}(3.67) + \log_{10}(10^2) = $ something $+ 2$.
If "something" is less than 1, then the floor of the entire expression, will be 2.
Since $1 < 3.67 < 10$, the log will be between 0 and 1.
So the "something" doesn't matter, taking the floor obliterates it.
 
@DavidWheeler You just made a huge rectangle.
 
Now we have to "add 1" because the "third digit place" is hundreds, and we start with "ones" ($10^0$)
@ABeautifulMind Wait, what?
 
@DavidWheeler Just look and you will see.
 
Look...where?
 
Sorry, I am too obscure. The huge chunk of your text in chat.
 
7:12 PM
At the block of writing :-)
 
Henceforth, I should request a chat basis of order 3, in order to minimize content.
Of course, only two dimensions will be visible here.
 
Please don't pass your 3 dimensional objects through our 2 dimensional world :-)
 
Ok... Thank you!! @DavidWheeler
 
I try to much, I might when forget.
Damn. Sorry about that.
I will try not to overly much, but I still might when I forget.
 
Hi everyone!
do you have any idea for:
0
Q: Greatest Commom Divisor

user159870In my lecture nores there is the following: $a, b \in \mathbb{N}, gcd(a, b)=d \Rightarrow \exists s, t \in \mathbb{Z} : sa+tb=d \Rightarrow tb \equiv d \pmod a$ If $gcd(a, b)=1$ then $tb \equiv 1 \pmod a \ $ $ \ \ \ [b]_a \in \mathbb{Z}_a^{\star}$ and $[b]_a^{-1}=[t]_a$ $$r_0=a, r_1=b \\ r_...

 
7:25 PM
@robjohn Theoretically, one can use the contour integration for compution this one too $$\int_0^{\pi/2}\frac{x^n\log{\sin{(x)}}}{\sin(x)}\,dx$$
 
Greetings
 
@infinitesimal Hi
 
How are you @Chris'ssis?
 
@infinitesimal Not that bad, thanks. You?
 
@Chris'ssis fine thanks.
 
7:28 PM
@robjohn And related to $$\int_0^{\pi/2}\frac{x \log{\sin{(x)}}}{\sin(x)}\,dx$$ it should also be interesting to study the case $$\int_0^{\pi/2}\frac{\sin(x)\log{\sin{(x)}}}{x}\,dx$$ where I only swapped $x$ and $\sin(x)$.
@infinitesimal Great. :-)
 
A person says "good morning" and exactly 12 hours later he says "good morning" again. How is that possible?
 
@robjohn The same rectangle contour should work nicely.
 
@Chris'ssis May I hope that one day your book will be entitled "Integrals of rational expressions of transcendental functions"?
 
Both statements being true at the time he says them.
 
@DavidWheeler I cannot promise you anything for the future. :-)
 
7:33 PM
When I learned some functions did not have primitives, I lost interest in the general case. I see you are much braver :)
 
@robjohn at the same time, one can also go the real analysis by using the substitution $-\log(\sin(x))=y$.
@DavidWheeler Well, I simply like what I do. That's all. :-)
@robjohn No, that contour doesn't work ... (actually not in that plane)
Let me check another version ...
 
@Chris'ssis I think you consider it a "fun game", like being good at chess. You'd probably make a great engineer, but it might not be as enjoyable for you.
 
@DavidWheeler I like engineering pretty much, I attended such a job in the engineering area. I might return to such a place one day.
 
@DavidWheeler I found the following:

$| \mathbb{Z}_n^{\star} |=\phi(n)$

We need $\log n$ digits to represent the elements of the group.

So, does it stand that $\lfloor \log_2 \phi(n) \rfloor +1=\log n$ ??
 
For me, I enjoy the change-of-variable theorem, as a theorem, but as for particular substitutions, not so much.
@MaryStar I think you're mixing apples and oranges
The totient function is "unpredictable" (having to do, as it does, with the distribution of prime integers, or more properly, prime factors of integers)
The number of digits required to represent the integers mod n, has only to do with n.
But the number of UNITS can be much smaller. I'm not sure where the base 2 log is coming from, either.
Usually the same number of digits is used for representing the units, as in the integers mod n. This isn't necessarily optimal.
 
7:50 PM
Hi @DavidWheeler
 
@BalarkaSen Hi?
 
I see that you finally hit 2K.
 
@Chris'ssis @robjohn are these integrals known ? $\int_0^{\pi } \cos ^{2 n}(x) dx$
 
@LeGrandDODOM Yes, they are.
2
 
@robjohn I meant even indices
 
8:01 PM
@LeGrandDODOM Well-known ...
 
@LeGrandDODOM $W_{2n}$ more or less ?
 
In mathematics, and more precisely in analysis, the Wallis' integrals constitute a family of integrals introduced by John Wallis. == Definition, basic properties == The Wallis' integrals are the terms of the sequence defined by: or equivalently (through a substitution: ): In particular, the first few terms of this sequence are: The sequence is decreasing and has strictly positive terms. In fact, for all : , because it is an integral of a non-negative continuous function which is not all zero in the integration interval (by the linearity of integration and because the last integral i...
 
@LeGrandDODOM You should know since from your profile pic I guess that you did Math Spé :P
 
@BalarkaSen I did. Toyed with the idea of losing rep by madly editing things to humorous effect, but meh. Too lazy.
 
@Chris'ssis Waiit. he wrote from $0$ to $\pi$ :O Doesn't that make 0 ?
 
8:03 PM
hah. seems like a good idea for timepass.
 
@ɧɿρρԹʅȝՇԵՐՎԾՌ enough
 
@ɧɿρρԹʅȝՇԵՐՎԾՌ And? For even indices?
 
@Chris'ssis -_- sorry
 
@ɧɿρρԹʅȝՇԵՐՎԾՌ :D
 
@Chris'ssis That is still $2W_{2n}$ then
 
8:05 PM
@robjohn It looks like that an integral here doesn't survive more than a few seconds ... :-)
@ɧɿρρԹʅȝՇԵՐՎԾՌ The idea is that one can reduce it to $0, \pi/2$ interval (and hence the Wallis' result.)
 
Yeah
@Chris'ssis What did he mean by 'enough' though ?
 
@ɧɿρρԹʅȝՇԵՐՎԾՌ Not sure.
 
Have the Dehn functions of most of the groups that you first meet when studying group theory been computed?
 
You're studying geometric group theory, @Alyosha?
As to the answer, almost any group is Gromov hyperbolic [*], and Dehn function of Gromov hyperbolic groups are known...
And of course, that applies to finite groups, which are trivially quasi-isometric to a point.
[*] : Theorem by Gromov - any given group is hyperbolic with probability 1.
 
I added a new question (after some time)
1
Q: Calculate $\int_0^{\pi/2}\frac{\sin(x)\log{\sin{(x)}}}{x}\,dx$

Chris's sisInspired by a question I saw these days, I try to calculate in closed form $$\int_0^{\pi/2}\frac{\sin(x)\log{\sin{(x)}}}{x}\,dx$$ So far no fruitful idea that is worth sharing. What way would you propose? Note I prefer ways, not necessarily solutions, but I have nothing against any of the opti...

 
8:20 PM
@Chris'ssis nice one. You already posted it in the chat though :p
 
@ɧɿρρԹʅȝՇԵՐՎԾՌ Yeap.
 
Given that most groups you study when first studying group theory, are finite, abelian, or hyperbolic, then yes they have been computed.
 
@BalarkaSen Do you have a link to that theorem?
 
Google. It's a famous theorem.
That's in fact the beginning of the theory of random groups, really.
 
Neither 'Gromov Theorem' or 'Gromov almost all groups are hyperbolic' yield any results.
Well, results that seem like that.
 
8:31 PM
Look up random groups
 
Thanks.
 
hello, please have you an example in $\R$ a compact set $A$ and a closed set $B$ such that $d(A,B)$ is not achived
please
 
@PaulPlummer I trust you're a geometric group theorist, then?
I am curious, as most of the people I talk to about mathematics here are, funny enough, geometric group theorists.
 
@BalarkaSen Not yet :), I am going to grad school next semester and GGT is one of the things I am looking to focus on.
 
8:42 PM
Oh, cool. I personally know nothing about geometric group theory, only stuff I have picked up from talking to a few people around me.
 
Hi!! How do we find limits like $ \lim_{x \to \infty} \frac{\sqrt(x^2-1)}{x} $ ? I tried using L'Hopital but it doesn't seem to work...
 
It seems like great stuff.
 
Do you know where you're going, @PaulPlummer?
 
-1
Q: measurement problem

sineadTwenty sweets weigh 150g.what is the weigh of one sweet?

:/
 
@MikeMiller Not yet
 
8:46 PM
come on guys, don't ignore me.. it's a simple question and I have a mid term
 
@hb20007 Have you tried the squeeze theorem?
 
@Paul You're familiar with Cannon and Thurston's conjecture on boundaries of hyperbolic groups?
 
@DavidWheeler no, but I shall look it up!
 
See if you can bound what's "under the radical" by two squares
 
Good luck on results, @PaulPlummer, if you haven't gotten them yet
 
8:50 PM
@ɧɿρρԹʅȝՇԵՐՎԾՌ Ironically, there's not enough information given to solve the problem; it's not specified whether the twenty sweets are all identical
 
@MikeMiller thanks, @balar
 
@teadawg1337 ( ͡° ͜ʖ ͡°)
 
@BalarkaSen I have heard of it and have been wanting to take a look at cannons paper on finite subdivision rules but havent done so yet
 
We can find the average weight, though :)
 
@DavidWheeler @teadawg1337 so all the sweets are 0 grams except two, one is 360 grams and the last one is -210 grams :D
 
8:53 PM
LOL
Ugh, that original tag though..... Measure theory, really?
 
@PaulPlummer yeah, you should read up about it. it's an interesting problem (i'm biased, because i know the guy who proved it for pairs $(H, G)$ with $H$ normal in $G$). only recently they found a counterexample to it, i think.
 
I would argue that a 0-gram sweet isn't sweet at all. And I would keep that -210 gram sweet safely locked up in the cloud chamber.
@teadawg1337 How long before measure theory gets a question as to which brand of ruler is the best?
 
cringes
 
Obviously, Lebesgue rulers can measure more things.
 
@ɧɿρρԹʅȝՇԵՐՎԾՌ are you done with my question? :-)
 
8:59 PM
@Chris'ssis The one with nested roots ?
 
@DavidWheeler Infra-Planck rulers are master race
 
@ɧɿρρԹʅȝՇԵՐՎԾՌ Also that one. I was mainly referring to the one posted on main. :-)
 
@Chris'ssis Oh sorry, I don't really have time at the moment :/
 
@ɧɿρρԹʅȝՇԵՐՎԾՌ OK
 
Wait, the nested radical that I solved a week or two ago?
Or is this a different nested radicals problem @Chris'ssis @ɧɿρρԹʅȝՇԵՐՎԾՌ
 
9:02 PM
My ruler doesn't have a Planck scale :(
 
@BalarkaSen Nice. They found a counterexample to the conjecture?
 
@teadawg1337 You didn't solve, you only guessed the answer. Pls don't say it. :-)
 
I had one once, but every time I looked at it, the marks moved.
 
I did too solve it :(
 
@teadawg1337 OK
 
9:03 PM
@PaulPlummer Yeah. It's a recent result.
 
@Chris'ssis which one is that?
 
@robjohn One I told you about many times. It was posted on dropbox.
 
@Chris'ssis okay. There have been so many nested radical problems, that I don't remember which was said when
 
I clearly remember solving a nested radical of yours, @Chris'ssis, I remember that I was the first to solve... Am I remembering incorrectly??
 
@teadawg1337 You only guessed the answer as far as I know.
 
9:06 PM
Yeah, that's the one that I solved...
Well you wouldn't want me to show my work here, would you???
 
@teadawg1337 OK, you solved them all.
 
Was that interpreted as blackmail? D:
Not my intention at all...
 
@teadawg1337 hehe, no worry, I didn't think of that. ;)
 
If you'd like, I could put my work on a private link and send the link your way @Chris'ssis
 
@teadawg1337 Heya
 
9:14 PM
@teadawg1337 I believe you, no need for efforts. ;)
 
Hello @N3buchadnezzar
 
@teadawg1337 Got some time to help me solve something silly? :p
 
Sure, lol
 
A man is a distance $s$ from his home, and has his loyal dog with him. He walks home with a speed of $v_m$, and his dog runs home at a speed of $v_d$.
When the dog returns, he runs back to his man and so forth.
@teadawg1337 I am trying to find the times $t[n]$ when the dog is home. My goal is try to try to write a parametric equation that gives the dogs position at time $t$. any ideas? :p
 
I know the total distance traveled can be modeled by a geometric series, but that's about it
 
9:19 PM
@teadawg1337 So $t[1] = d / v_d$ obviously right?
 
Are we assuming no acceleration? And yes @N3
 
@teadawg1337 Yeah, no acceleration, no drag no friction, no silly air resistance and no aliens.
 
Hmm...
 
@teadawg1337 So from the "origo" and back to the owner, should be the intersection $$ v_d( t[2] - t[2]) = d - t[2] \cdot v_m $$
this gives me a correct $t[2]$, but does not work for $t[4]$ :p
 
Let me give it a go
Definitely more challenging than it appeared at first glance, @N3buchadnezzar
 
9:25 PM
@teadawg1337 :p
@teadawg1337 Be glad the aliens and accelaration stayed out of this
 
I'm getting a far nastier expression for $t_2$...
 
@robjohn I decided to see the doctor and start meds again.
 
Oh, back to the owner
I've got $t_3$ in that case, and it's ugly (maybe I did something wrong)
 
@ABeautifulMind Jasper, my prayers are your doctor proves to be a good one.
 
@DavidWheeler I am not restarting therapy yet, because I am not ready to work on those issues with a therapist, but I will when the time comes.
I did not really like my previous therapist, I might try to see someone else.
 
9:38 PM
Well, even just a doctor who gives a prescription has to be sensitive to how well it works for you. Proper dosages can take time to figure out.
 
I am meeting one of my professors for lunch next week. He has a son who is a therapist. I might see him instead.
 
@teadawg1337 Bah
 
For example, for a while I took this one medication that helped with depression, but dulled my mind-things didn't interest me.
 
user129943
Hey guys
 
user129943
WTF
 
9:41 PM
I found, by trial-and-error, that 1/4 the dose worked LOTS better.
 
@DavidWheeler LOL
 
I didn't get too depressed, and still felt alive.
 
user129943
Okay I cannot log out - I don't really want to clear my cookies.
 
user129943
Hey guys it's Alec Teal here
 
Mmm. cookies....
 
9:43 PM
I did a numerical test if you let $d = 10$, $v_d = 6$ and $v_h = 4$.
Then $t[1] = 5/3$, $t[2] = 2$, $t[3] = 7/5$. do you get the same?
 
user129943
Anyway I need a favour, two people really helped me on a question that didn't get much attention. (especially the guy who didn't get my tick) math.stackexchange.com/questions/1172594/… can you +1 the answers here please - they really helped me and I don't like that the one guy only got +10 for it
 
@I'mGettingThere Points don't matter.
 
user129943
@ABeautifulMind to the guy with 277k - yeah you're right, but the other new guy. It's wrong that an easy real-analysis answer will earn ya loads but this - gains only 10.
 
$\displaystyle t_3=\frac{d-v_m{\frac{d}{v_d}}-{\frac{v_m{d}-\frac{v_m^2{d}}{v_d}}{v_m+v_d}}}{v_‌​d}+\frac{d-v_m{\frac{d}{v_d}}}{v_m+v_d}+\frac{d}{v_d}$
Pretend the first denominator is $v_d$
 
user129943
math.stackexchange.com/a/1172936/66223 the guy who posted this answer has <200 rep, he's new, he gave me a great answer. Please reward that
 
user129943
9:47 PM
That's a direct link to the answer, can't make it easier than that!
 
user129943
Anyway how is everyone?
 
@teadawg1337 I can not be bothered plugging in. If you do not get $7/3$ you maed a mistake (it should be 7/3 approx 2.33... and not 7/5)
 
The "basic set-theoretical definitions" of sets mathematicians commonly employ can get pretty horrendous. Logically airtight, sure, but lacking a certain transparency.
 
user129943
Thanks the 2 of you that upvoted that answer
 
@N3buchadnezzar I get $\frac{7}{3}$
 
9:52 PM
@teadawg1337 Ey
 
If you want to find a parametric equation for THAT
Then...
 
@teadawg1337 I think you can simplify it down to $$ t[3] = \frac{4s}{v_m + v_d} - \frac{d}{v_d} $$
Hard to find a pattern for t :p
 
I thought it was originally $v_m$ and $v_d$?
 
@teadawg1337 Fixed, I used a slightly different notation in my notes.
 
Oh, it's $s$ instead of $d$
 
9:55 PM
@teadawg1337 but I think I managed to make an reccurence relation for it
 
Hmm...
I can't see how to simplify my expression down
 
@teadawg1337 $$ \begin{align*} v_d \cdot ( t[n] - t[n-1] ) & = s - v_m \cdot t[n] \\ t[n-1] & = 2\cdot t[n-2] - t[n-3] \end{align*}$$
 
I meant how did you get to $\displaystyle \frac{4s}{v_m+v_d}-\frac{s}{v_d}$
 
@teadawg1337 Maths
 
My brain is dead today, give me a sec to verify that
 
10:02 PM
@teadawg1337 I did it likethis
 
What the hell is $v_g$?
 
@teadawg1337 $v_g = v_h = v_d$, just assume it is the speed of the dog unless written $v_m$ :p
 
Wait, why are there three velocities?
...
 
I am terrible at sticking to one notation..
 
Oh, $v_g$ and $v_h$ have the same magnitude but opposite direction(?)
nvm, it's irrelevant lol
 
10:06 PM
I think i fixed the notation now
 
10:18 PM
@N3buchadnezzar Oh, it can be simplified to that...
 
@teadawg1337 :p
 
Hello @DanielFischer!!!
I am given some recurrence relations $T(n)$ and I have to give asymptotic upper and lower bounds for $T(n)$. We assume that $T(n)$ is continuous for $n \leq 2$.
How can we use the fact that $T(n)$ is continuous for $n \leq 2$?
 
@teadawg: You tell 'im!
 
@TedShifrin ello
 
re salut @Hippa
 
10:33 PM
@TedShifrin What do you think he meant here ? chat.stackexchange.com/transcript/message/20351338#20351338
 
Hello @ted.
 
rehi Jasper
 
@TedShifrin: My area was downgraded to a winter weather advisory, though I live a couple of miles south of the county line. The county directly north has a winter storm warning, so idk what to expect
 
Je ne sais pas, @Hippa. Maybe he doesn't want you criticizing/meddling.
Cross your toes, @teadawg.
 
@TedShifrin I answered his question though q_q it was in fact $2W_{2n}$, quite clearly..
Isn't he really from polytechnique ?
 
10:36 PM
Not so far as I know, @Hippa.
 
Ah. That might be it q_q
 
:20352352 yes, I remember doing that one.
 
However, looking at the questions he answered, he should be way good enough
 
Except for chris'ssis, @Hippa, we're not about competition.
 
@robjohn I did the asky thing again
 
10:39 PM
What do you mean @TedShifrin (I mean, what's the link ?)
 
@teadawg1337 Meh I just asked, will see what people say tommorow
 
The way he told me enough, I took it as if someone was asking me without any bad intent 'haven't you seen this/that at school' and I was answering don't question me !, it feels weird
 
You were sounding like it should be trivial to someone of his situation, @Hippa, so perhaps he took offense.
 
0
Q: I will pay someone to help with discrete mathematics

AshelyI will pay for someone to help me with a math assignment. I could not think of a better place to find someone to help me. I would like to skype or use google hangouts to have someone aid me in an assignment. Want to make some extra money?

:/
@TedShifrin Oh ok
 
@ɧɿρρԹʅȝՇԵՐՎԾՌ Isn't that person paying a professor to do that exact thing? Why are students afraid of their teachers?
 
10:44 PM
@DavidWheeler That's a good question. I don't have a clue, since I spam my teachers with questions :3
 
college professors aren't as receptive as (good) high school teachers, @Hippa.
 
Better still, why do students wish to fool their teachers into thinking they "get it" when they don't?
 
I have had good high school teachers, and have good college professors too :D
 
I'm so frustrated. I told my students to expect a problem on the exam where they'd have to (as in homework) use the contraction mapping principle to prove that $f(x)=y$ for $y$ in a certain domain has a root. And almost no one is getting it. Very upset.
 
I suppose some of it has to do with colleges functioning as "filter mechanisms" rather than centers of learning.
 
10:46 PM
@TedShifrin What was the exercise exactly ?
 
@Hippa: Consider $B=B(0,4)\subset\Bbb R^n$. Suppose $f:B\to\Bbb R^n$ is $C^1$ and $\|Df(x)-I\|\le 1/3$ for all $x\in\bar B(0,3)$. Suppose, moreover, that $f(0)=a$ and $\|a\|\le 1$. Prove that if $\|y\|\le 1$, then there is a unique $x\in\bar B(0,3)$ with $f(x)=y$.
ugh ... too many macros
 
urm what's $B(0,4)$ ? (to me that's the centered open ball of radius 4)
 
damn, that was way too stressful.
open ball centered at $0$
 
What's this about balls, lol.
 
And what is $D$ ?
 
10:50 PM
derivative
behave, Jasper
@David: But I'm teaching the top 0.1% of the students in this class.
Hmm, probably 0.01%
 
I'm in top 105% :D
 
how accomplished of you, @Hippa ... I expect a solution in 10 minutes.
 
Probably between 103 and 105 % :3
 
@TedShifrin I don't know what that means
 
@David: The students in my class thrive on being challenged and learning, but they just have gone through life with classes and tests being a formality.
 
10:52 PM
By "class" do you mean "year"?
 
No, I mean a particular class of 15-20 I'm teaching.
 
If you're only teaching 0.1% of 20 people, you're for all intents and purposes wasting time.
 
There was once an economics student who took a group theory class and he could do problems that nobody else could.
 
No, no ... I'm sorry. The percentage was of the total undergraduate body. My stupidity.
Economics students are often quite mathematically talented, but underchallenged in economics :P
 
i still remember my first math class in college. i had aced every class i took up until then, and got AP credit, so jumped right in to vector calculus. i nearly flunked the first quarter.
 
10:55 PM
where was that, @David?
 
I went to Reed College
 
ah, fun
 
I put on 10 kg and now I dropped 10 kg.
 
when I went to college, that was the location for all the hippy types :)
 
The light bulb moment for me was $e_1 = \mathbf{i}$ etc.
 
10:57 PM
I start my course with definition of vectors, addition, subtraction, etc.
 
Yeah, well it's all fun and games until the Implicit Function Theorem.
 
LOL ... that was this exam.
I didn't know Reed had such a course for freshmen a while ago.
 
People are like, wait, what? Why do we need the Jacobian (determinant) to be non-zero in a neighborhood?
 
@TedShifrin I've never seen the contraction mapping th. before, so just to be sure, is it the following : if $f$ is a contraction mapping ($\exists a<1,\forall x,y,|f(x)-f(y)|\le a|x-y|$) then it has a unique fixed-point on a given metric space ?
Sounds like lipschitz stuff to me
 
$q=a$, @Hippa. In our case, closed subset of $\Bbb R^n$. In your case, complete metric space.
 
11:00 PM
I believe it's the Banach fixed-point theorem (or so my rusty memory says-it's been known to lie)
 
@Hippa not quite
 
No, Lipschitz doesn't require contraction at all.
 
What's the difference with a contraction mapping and a q<1-lipschitz mapping ?
 
aucune @Hippa
 
11:03 PM
I think what Ted means is contraction is a stronger condition than Lipschitz (guess)
 
@ɧɿρρԹ I will contrinue that tomorrow ; right now I'm too tired for that, I'm going to bed. Good night.
 
Bonne nuit, @Ramanewb
 
Bony knights!
 
@TedShifrin Is there some kind of trick you have given your students beforehand (or some additional knowledge/whatever I might not know) ?
 
11:05 PM
good night @ted
 
Sure, @Hippa. We spent over a week on this stuff and using it to prove the inverse function theorem. So one needs to set up a contraction map $\phi$ so that $\phi(x)=x \iff f(x)=y$.
 
@TedShifrin Ugh I figured out that much q_q
 
Well, so what's your candidate for $\phi$, @Hippa?
 
Obama ?
Let's get serious, ok :D
@TedShifrin That's the whole question lol.
 
I think Obama supports HIPPA, so the reciprocity is understandable
 
11:08 PM
Let me search a bit longer
 
@Hippa: The point is not to search. It's to think.
It's not difficult to invent a function $\phi$ with that property.
 
@TedShifrin How do you search without thinking q_q
 
Hint: $f(x)=y \iff y-f(x)=0$.
 
My problem is with having a contractive $\phi$
 
Well, the norm condition is relevant.
 
11:11 PM
Woah, is this vector calc-type stuff? I need a serious refresher...
 
You haven't learned this stuff, @teadawg, but you will if you try to do my course.
 
I would still need "brushing up"...I've forgotten so many things....
 
@TedShifrin Oh wait... we haven't defined $y$
 
@Ted My coursework is getting progressively more time-consuming, I still try to fit in a couple of lectures every few days
 
It can't be $\phi(x)=x \iff f(x)=y$ for all $x,y$
 
11:14 PM
no, @hippa. Fix $y$ with the conditions as in the problem.
 
Gosh I need to write things clearly
I didn't write the problem as you did xD hence my messing up
 
C'est pas ma faute!
 
An obvious (may be non contractive) solution would be like $\phi(x)=x+y-f(x)$
 
well, try it
 
17 credit hours was a bad idea...
 
11:16 PM
$|\phi(x)-\phi(z)|=|x-z-f(x)+f(z)|$
 
You need the mean value inequality, @Hippa (the derivative norm condition)
 
Oh good idea
f(b)-f(a)=(b-a)f'(c) is the equality... I can't remember the inequality
 
inequality with vector functions ...
 
Should be simple though :/
 
no equality for maps $\Bbb R^n \to \Bbb R^m$
 
11:18 PM
I don't think I've seen that
 
well, @Hippa, you have a few things left to learn
 
more than a few :-)
where can I find that ?
The wiki article on the MVT gives the one I wrote above
 
try mean value inequality
 
Mean value theorem for vector-valued functions maybe
 
or see my appropriate lecture, @Hippa .. if you're not too busy making memes.
 
11:20 PM
$f(x+h)-f(x)=\int_0^1f'(x+th)dt*h$ ?
 
no, $\|f(b)-f(a)\|\le \max \|Df(c)\| \|b-a\|$ for $c$ chosen on a path from $a$ to $b$.
 
Oh I remember now :D
yeah it's quite logical
 
you can report to me later when you've solved it, @Hippa.
Dinnertime and have to finish grading exams.
 
ok
I'm just annoyed that $\phi'=1-f'\neq x-f'$
 
tumbleweeds
 
11:55 PM
Hola
 
'Owdee
 
Fine!
 
Sorry. Idk what that means
 
Which?
 
11:59 PM
"'Owdee"
 
'Owdee is just a corrupt spelling of "howdy" which is a rough approximation of "hola" in the southwestern united states
 
Oh. Haha. Sorry. Hello
 

« first day (1672 days earlier)      last day (3332 days later) »